What is Magnetic fied: Definition and 33 Discussions

No Wikipedia entry exists for this tag
  1. A

    Derivation Of Torque On Current Loop Due To Uniform Magnetic Field

    I can derive it for a circular loop: $$dF=BI\sin\phi\ dl=BIr\sin\phi\ d\phi$$ Torque on quarter circle when field is parallel to plane of loop=$$\tau=\int^{(\pi/2)}_0 BI \ dl \sin\phi (r\sin\phi)$$$$=\int^{(\pi/2)}_0 BIr^2 \sin^2\phi\ d\phi$$ Net torque=##4\tau=BIA## If magnetic field is at any...
  2. V

    Why are the effects of a magnetized material neglected in this case?

    According to what I know, the net magnetic field ##\vec B## inside a solenoid is given by $$\vec B = \mu_0 \vec H + \mu_0 \vec I$$ where ##\vec B## is the net magnetic field inside a current carrying solenoid, ##\vec H## is the magnetic intensity ( aka magnetic intensity or magnetizing field...
  3. patric44

    Finding the magnetic field B given the vector potential A ?

    hi guys this seems like a simple problem but i am stuck reaching the final form as requested , the question is given the magnetic vector potential $$\vec{A} = \frac{\hat{\rho}}{\rho}\beta e^{[-kz+\frac{i\omega}{c}(nz-ct)]}$$ prove that $$B = (n/c + ik/\omega)(\hat{z}×\vec{E})$$ simple enough i...
  4. gumthakka

    Current and stationary charges

    I've learned that moving charges produce magnetic fields which in turn affect other charges in motion. After seeing explanations that point to special relativity, I am kind of confused. Can **ALL** magnetic fields be accounted as some kind of electric field from a particular reference frame...
  5. Elder1994

    Magnetic field due to the current flowing in a bent wire

    Hello, in this problem I'm supposed to calculate de magnetic field due to a bent wire at any point of the x-axis after the bending of the wires. It is obvious that the part of the wire that is parallel to the x-axis makes no contribution to the field so we can focus on the other part of the...
  6. A

    What is Current? I know it is a scalar but I found something weird....

    While I was going through "Introduction to Electrodynamics" by David J. Griffith I see the line "Current is a vector quantity". But we know it doesn't obey the vector algebra (addition ). Then how it can be a vector?... Please help me
  7. M

    The right hand rule in a magnetic field

    Homework Statement An electron moving in the -z direction enters a region that has a uniform magnetic field in the +y direction. When the electron enters this region, what direction will it be deflected in? Homework Equations F=qv X B The Attempt at a Solution I tried answering this question...
  8. P

    Quantum Locking and the Earth's Magnetic Field

    Don't know if this is the right place to post this, but it seemed like the best place to start. I'm wondering how much stronger would the Earth's magnetic field need to be in order to make quantum levitation using Earth's magnetic field alone a possibility. Bonus: Effects the change in strength...
  9. T

    Simple Question about the direction of magnetic induction

    Homework Statement There is a magnetic field B=0.02 Tesla and its direction is toward the plane (perpendicular to plane, shown by X X X X X - I'm not sure how to say this direction in English,sorry). we have a copper wire (uniform bar) perpendicular to magnetic field from south to north. we...
  10. H

    Electric Force on a Charge in a Solenoid

    Homework Statement [IMG]http://[url=https://ibb.co/dgUy6T]https://preview.ibb.co/iyqS0o/20180525_213806.jpg Since i only know the field direction, increasing go into page. Why the answer is C? Why the answer "a" ? The R and r on the pic is respected to what? Homework EquationsThe Attempt...
  11. A

    Field of a circular loop around its axis issue

    Good , i want to derive the formula for field of a circular loop around its axis but i was stucked with the following point: they consider dl perpendicular to r ! why is that?? many thanks in advance!
  12. peroAlex

    B-Field on Axis of Equilateral Triangle

    Homework Statement I came across a pretty interesting question that asks for magnetic flux density (B-field) on the axis of the equilateral triangle. This axis is meant to be perpendicular to triangle's surface passing through its centroid. Assuming that a triangle has sides denoted ##a## and...
  13. A

    What is a magnetic field "made" of?

    I have looked this up on several occassions and never satisfied. It seems that either the answer keeps repeating magnetic or electromagnetic field over and over. Or the answer discuss the properties of electromagnetism. Is the field a wave/particle like a photon ? Is it emitted like a photon...
  14. D

    Could a distribution of magnetic Dipoles create an Energy minimum

    The think is that I would like to create a magnetic trap "minimum of magnetic potential energy in all 3 directions of space" using a distribution of magnetic dipoles as a field generator. In other words, I would like to know if setting in some way some magnetic dipoles is possible to create a...
  15. Asmaa Mohammad

    The magnetic flux and the induce emf in the dynamo.

    Homework Statement Multiple-choice question: Homework EquationsThe Attempt at a Solution I know that the induced emf is maximum when the coil is parallel to the direction of the magnetic flux, so I should choose either B or C, and I have read the the rate by which the coil cut the magnetic...
  16. peroAlex

    Flux and Current of Magnetic Core With Air Gap

    I am a student of electrical engineering. This task appears in our textbook. However, there were no solutions provided. I tried understanding the task but got lost in the process. This is why I seek help here on this site. I would like to ask you for guidance and any sort of advice on how to...
  17. G

    Can a time-varying magnetic field pass a metal sheet?

    Hello. I'm using CT (Current Transformer) to measure the current flowing on the power line. The frequency of the current is 13.56 MHz. CT appeared to be metal-shielded (This metal case of CT may be grounded when CT is used). CT has a toroidal shape and the power line under the measurement...
  18. 1

    Help designing/finding a vendor for this magnetic field

    I need help find a electromagnet suitable for this http://imgur.com/a/vkm4T Each electromagnet (the 3 in the center) needs a repelling force of about 20 to 40lbf. I stupidly bought these from amazon http://a.co/aSftIat. Its a holding electromagnet so the N and S poles are essentially on the same...
  19. H

    Magnetic field in a circular loop

    Homework Statement A conductor consists of a circular loop of radius R and two long, straight sections as shown in the figure. The wire lies in the plane of the page and carries a current I. (a) What is the direction of the magnetic field at the center of the loop? (b) Find an expression for...
  20. KSNphysics

    Physics Magnetism - Wire Carrying Current in a Loop

    Homework Statement This is a theoretical question, one I'm not sure about. It just randomly popped into my head. Say there is a jumbled wire, one in a glob shape but not overlapping. (Like a deformed circle, or maybe just a rectangle). Assuming we are in a place called "physics utopia" where...
  21. Milind Chakraborty

    A Charged Particle moving in Uniform Magnetic Field

    Homework Statement Problem given in the image attached. Uniform Magnetic Field : B Positive Charge : q Uniform Velocity : v Mass : m Charged particle enters the magnetic field making an angle θ with the plane perpendicular to the magnetic field. Width of the region of Magnetic field : d d <...
  22. DavideGenoa

    I Magnetic field by infinite wire: convergence of integral

    Let ##\boldsymbol{l}:\mathbb{R}\to\mathbb{R}^3## be the piecewise smooth parametrization of an infinitely long curve ##\gamma\subset\mathbb{R}^3##. Let us define $$\boldsymbol{B}(\boldsymbol{x})=\frac{\mu_0...
  23. S

    A Can Fluctuation-Dissipation Theorem Apply to Magnetic Forces

    Let's say I have multiple spin systems (atoms in a protein) in a solution of water and the spin systems are all producing a magnetic field \mathrm{B_{loc}} that affects nearby spin systems. Will the fluctuation-dispersion theorem apply to the force generated by a spin's magnetic field...
  24. DavideGenoa

    I Integral: magnetic field inside infinite cylindrical current

    Let ##V\subset\mathbb{R}^3## be an infinitely high solid cylinder, or a cylindrical shell of radii ##R_1<R_2##, whose axis has the direction of the unit vector ##\mathbf{k}##. For any point of coordinates ##\boldsymbol{r}\notin \bar{V}## external to ##V## the Lebesgue integral (which is...
  25. C

    How to Optimize a simple Electromagnet

    My goal is to make an electromagnet that delivers 100mT magnetic field on one end of the iron. The iron core has a diameter 2.0mm and length 0.1m. I am using a copper wire that can carry 2.3A of current, and I am using a DC power supply. I have been able to create a 100mT magnetic field on the...
  26. T

    Skin effect and magnetic field of a flattened conductor?

    A conductor with a large high frequency AC current causes a "Skin Effect" meaning that most of the current flows at the surface of the conductor. What does the skin effect look like if you have a conductor with one side flattened? And what does the resulting magnetic field look like? See...
  27. L

    Magnetic Field in parallel plate capacitor

    Homework Statement A parallel plate capacitor of area 60cm^2 and separation 3mm is charged initially at 90 micro coulomb. If the medium between the plates get slightly conducting and the plate loses charge initially at the rate of 2.5 *10^-8 C per sec then what is the magnetic field between the...
  28. E

    Magnetic hysteresis loop area meaning

    Let's consider the Magnetic hysteresis loop of a certain material: https://www.nde-ed.org/EducationResources/CommunityCollege/MagParticle/Physics/HysteresisLoop.htm is an example. In many sites and books it is written that its area is proportional to the energy wasted as heat, so A = kE_d. In...
  29. E

    Calculate flux in a ferrite bead on a wire

    Suppose that a ferrite bead is put around a cable where a constant current I flows, just like in this image. The coordinate system has the z axis along the cable. Let's evaluate the current through the (x,y) plane: according to the Ampère's law, the only magnetic field component generated by...
  30. KronosZ

    Conservation of Energy and current

    Assume there is a table with an infinite long wire passing through the centre and few iron pieces lying around the wire on the table. Now I pass current through the wire which creates electro magnetic field due to which all the iron particles get magnetic potential energy. They are attracted...
  31. arpon

    Calculating Magnetic Field in a Square with Varying Resistances

    Homework Statement [/B] Consider a square whose successive sides of length L has resistances R, 2R, 2R, R respectively. If a potential difference V is applied between the points (call them , say , A and B) where the sides with R and 2R meet. Find the magnetic field B at the center of the...
  32. D

    Problem in Magnetic Field and magnetic effect of electric current

    I read in class 10. I am having problem understanding magnetic fields, magnetic flux,magnetic flux density,magnetic field strength,magnetic permeability and strength of magnetic field produced due to a straight and closed loop current carrying conductor at a certain point at a certain distance...
  33. S

    Magnetic fied at center of semicircle Ampere's Law Biot-Savart Law

    Homework Statement A long wire is bent into semicircle of radius R at its center and continues on to infinity in both direcions with the straight segments remaining parallel, as shown in the figure below. Use your knowledge of superposition, Ampere's Law, and the Biot-Savart Law to determine...
Back
Top